Homogeneous Linear DE's - solving IVP's

  • Thread starter Thread starter tatiana_eggs
  • Start date Start date
  • Tags Tags
    Homogeneous Linear
Click For Summary
The discussion focuses on solving the initial value problem (IVP) for the homogeneous linear differential equation d^2y/dt^2 - 4 dy/dt - 5y = 0, with conditions y(1)=0 and y'(1)=2. The general solution derived is y=c1e^(5t)+c2e^(-t). The user attempts to find constants c1 and c2 by substituting the initial conditions into the equations, leading to a system of linear equations. A mistake in calculating c1 is identified, where it should be c1 = e^(-5)/3 instead of e^(5)/3, which aligns with the book's solution of y = (1/3)e^(5(t-1)) - (1/3)e^(-(t-1)). The discussion highlights the importance of careful algebraic manipulation in solving differential equations.
tatiana_eggs
Messages
69
Reaction score
0
Homogeneous Linear DE's -- solving IVP's

Homework Statement



Solve the given IVP:

d^2y/dt^2 - 4 dy/dt -5y = 0; y(1)=0, y'(1)=2


Homework Equations



N/A

The Attempt at a Solution



I've solved and got the general solution y=c1e5t+c2e-t

I'm plugging in the following to solve for my two constants:

y(1)=0=c1e5+c2/e

y'(1)=2=5c1e5-c2e

So I have a system of 2 linear equations, and I can just add the two together and get:

2=6c1e5

and solving for c1 = e5/3

I would go on and solve for c2, but I checked the back of the book and they have:

y = 1/3e5(t-1)-1/3e-(t-1)

How did they get c1 = 1/3 and e5(t-1) ?
 
Physics news on Phys.org
Hi tatiana_eggs! :smile:
tatiana_eggs said:
2=6c1e5

and solving for c1 = e5/3

erm :redface: … c1 = e-5/3 …

which gives the 1/3e5(t-1) in the book. :wink:
 


Oh my gosh... duh.. I seem to be slowly losing my algebra skills as I learn more and more math.

Thanks so much!
 
Question: A clock's minute hand has length 4 and its hour hand has length 3. What is the distance between the tips at the moment when it is increasing most rapidly?(Putnam Exam Question) Answer: Making assumption that both the hands moves at constant angular velocities, the answer is ## \sqrt{7} .## But don't you think this assumption is somewhat doubtful and wrong?

Similar threads

  • · Replies 3 ·
Replies
3
Views
2K
  • · Replies 2 ·
Replies
2
Views
2K
  • · Replies 12 ·
Replies
12
Views
2K
Replies
4
Views
2K
  • · Replies 2 ·
Replies
2
Views
2K
Replies
7
Views
2K
Replies
7
Views
2K
Replies
3
Views
2K
  • · Replies 10 ·
Replies
10
Views
2K
  • · Replies 5 ·
Replies
5
Views
1K